LSAT and Law School Admissions Forum

Get expert LSAT preparation and law school admissions advice from PowerScore Test Preparation.

User avatar
 Dave Killoran
PowerScore Staff
  • PowerScore Staff
  • Posts: 5852
  • Joined: Mar 25, 2011
|
#59622
Complete Question Explanation
(The complete setup for this game can be found here: lsat/viewtopic.php?t=15933)

The correct answer choice is (A)

One approach to attacking this question is to use the hypotheticals from the previous three questions. Each contains scenarios that match the condition in this question stem, and collectively that information can be used to eliminate every incorrect answer choice. If you do not choose that approach, this question can still be done easily by making hypotheticals.

The condition in the question stem, in combination with the first rule, creates a powerful GLP block:

pt44_o04_g2_q12a.png
This block can only be placed on Monday-Tuesday or Tuesday-Wednesday:

pt44_o04_g2_q12b.png
Of course, due to the second rule, H cannot be placed on the same day as G, so H must be placed on Wednesday in the first hypothetical and on Monday in the second hypothetical.

In addition, because of the fourth rule, in the first hypothetical S must be placed on Monday (otherwise, if it were placed on Wednesday, then H could not be placed on Monday), and therefore K is placed on Monday in the first hypothetical. In the second hypothetical K and S form a dual option:
pt44_o04_g2_q12c.png
Answer choice (A): This is the correct answer choice.

Answer choices (B), (C), (D), and (E): As shown in the two hypotheticals, G, K, L, P, and S could be placed on Tuesday. Thus, each of these answer choices is incorrect.

Get the most out of your LSAT Prep Plus subscription.

Analyze and track your performance with our Testing and Analytics Package.